The concept of associate for finite abelian groups












1












$begingroup$


Let $G$ be a finite abelian group of order $n$, and let $a$ and $b$ be elements of $G$. If $a$ generates the same subgroup as $b$, must there be an integer $i$ prime to $n$ such that $ia=b$?



I can prove this when $G$ is cyclic, but not in general.










share|cite|improve this question









$endgroup$












  • $begingroup$
    Where did you encounter this problem?
    $endgroup$
    – Shaun
    Dec 10 '18 at 8:14










  • $begingroup$
    Hint: Consider the subgroup generated by $a$ (or $b$, as this is the same subgroup). You have shown that the claim holds in this subgroup. How could it not hold in the bigger group?
    $endgroup$
    – Tobias Kildetoft
    Dec 10 '18 at 8:22






  • 1




    $begingroup$
    @Shawn. The problem arises in the case that the decomposition of $G$ into a product of cyclic $p$-groups contains two or more factors of order a power of $p$. In the case that $G$ is cyclic this can't happen, and one proves the assertion by first handling the case that $n$ is a prime power (which is easy) and then applying the Chinese Remainder Theorem.
    $endgroup$
    – falang
    Dec 10 '18 at 8:26










  • $begingroup$
    @Tobias: The bigger group may have order divisible by primes that do not divide the order of $a$.
    $endgroup$
    – falang
    Dec 10 '18 at 8:29






  • 1




    $begingroup$
    @Shaun: Where did I encounter the problem? I noticed that it was true for $G$ finite cyclic and wondered if it was true for arbitrary products of finite cyclic groups.
    $endgroup$
    – falang
    Dec 10 '18 at 8:39
















1












$begingroup$


Let $G$ be a finite abelian group of order $n$, and let $a$ and $b$ be elements of $G$. If $a$ generates the same subgroup as $b$, must there be an integer $i$ prime to $n$ such that $ia=b$?



I can prove this when $G$ is cyclic, but not in general.










share|cite|improve this question









$endgroup$












  • $begingroup$
    Where did you encounter this problem?
    $endgroup$
    – Shaun
    Dec 10 '18 at 8:14










  • $begingroup$
    Hint: Consider the subgroup generated by $a$ (or $b$, as this is the same subgroup). You have shown that the claim holds in this subgroup. How could it not hold in the bigger group?
    $endgroup$
    – Tobias Kildetoft
    Dec 10 '18 at 8:22






  • 1




    $begingroup$
    @Shawn. The problem arises in the case that the decomposition of $G$ into a product of cyclic $p$-groups contains two or more factors of order a power of $p$. In the case that $G$ is cyclic this can't happen, and one proves the assertion by first handling the case that $n$ is a prime power (which is easy) and then applying the Chinese Remainder Theorem.
    $endgroup$
    – falang
    Dec 10 '18 at 8:26










  • $begingroup$
    @Tobias: The bigger group may have order divisible by primes that do not divide the order of $a$.
    $endgroup$
    – falang
    Dec 10 '18 at 8:29






  • 1




    $begingroup$
    @Shaun: Where did I encounter the problem? I noticed that it was true for $G$ finite cyclic and wondered if it was true for arbitrary products of finite cyclic groups.
    $endgroup$
    – falang
    Dec 10 '18 at 8:39














1












1








1


1



$begingroup$


Let $G$ be a finite abelian group of order $n$, and let $a$ and $b$ be elements of $G$. If $a$ generates the same subgroup as $b$, must there be an integer $i$ prime to $n$ such that $ia=b$?



I can prove this when $G$ is cyclic, but not in general.










share|cite|improve this question









$endgroup$




Let $G$ be a finite abelian group of order $n$, and let $a$ and $b$ be elements of $G$. If $a$ generates the same subgroup as $b$, must there be an integer $i$ prime to $n$ such that $ia=b$?



I can prove this when $G$ is cyclic, but not in general.







abstract-algebra number-theory






share|cite|improve this question













share|cite|improve this question











share|cite|improve this question




share|cite|improve this question










asked Dec 10 '18 at 8:04









falangfalang

309110




309110












  • $begingroup$
    Where did you encounter this problem?
    $endgroup$
    – Shaun
    Dec 10 '18 at 8:14










  • $begingroup$
    Hint: Consider the subgroup generated by $a$ (or $b$, as this is the same subgroup). You have shown that the claim holds in this subgroup. How could it not hold in the bigger group?
    $endgroup$
    – Tobias Kildetoft
    Dec 10 '18 at 8:22






  • 1




    $begingroup$
    @Shawn. The problem arises in the case that the decomposition of $G$ into a product of cyclic $p$-groups contains two or more factors of order a power of $p$. In the case that $G$ is cyclic this can't happen, and one proves the assertion by first handling the case that $n$ is a prime power (which is easy) and then applying the Chinese Remainder Theorem.
    $endgroup$
    – falang
    Dec 10 '18 at 8:26










  • $begingroup$
    @Tobias: The bigger group may have order divisible by primes that do not divide the order of $a$.
    $endgroup$
    – falang
    Dec 10 '18 at 8:29






  • 1




    $begingroup$
    @Shaun: Where did I encounter the problem? I noticed that it was true for $G$ finite cyclic and wondered if it was true for arbitrary products of finite cyclic groups.
    $endgroup$
    – falang
    Dec 10 '18 at 8:39


















  • $begingroup$
    Where did you encounter this problem?
    $endgroup$
    – Shaun
    Dec 10 '18 at 8:14










  • $begingroup$
    Hint: Consider the subgroup generated by $a$ (or $b$, as this is the same subgroup). You have shown that the claim holds in this subgroup. How could it not hold in the bigger group?
    $endgroup$
    – Tobias Kildetoft
    Dec 10 '18 at 8:22






  • 1




    $begingroup$
    @Shawn. The problem arises in the case that the decomposition of $G$ into a product of cyclic $p$-groups contains two or more factors of order a power of $p$. In the case that $G$ is cyclic this can't happen, and one proves the assertion by first handling the case that $n$ is a prime power (which is easy) and then applying the Chinese Remainder Theorem.
    $endgroup$
    – falang
    Dec 10 '18 at 8:26










  • $begingroup$
    @Tobias: The bigger group may have order divisible by primes that do not divide the order of $a$.
    $endgroup$
    – falang
    Dec 10 '18 at 8:29






  • 1




    $begingroup$
    @Shaun: Where did I encounter the problem? I noticed that it was true for $G$ finite cyclic and wondered if it was true for arbitrary products of finite cyclic groups.
    $endgroup$
    – falang
    Dec 10 '18 at 8:39
















$begingroup$
Where did you encounter this problem?
$endgroup$
– Shaun
Dec 10 '18 at 8:14




$begingroup$
Where did you encounter this problem?
$endgroup$
– Shaun
Dec 10 '18 at 8:14












$begingroup$
Hint: Consider the subgroup generated by $a$ (or $b$, as this is the same subgroup). You have shown that the claim holds in this subgroup. How could it not hold in the bigger group?
$endgroup$
– Tobias Kildetoft
Dec 10 '18 at 8:22




$begingroup$
Hint: Consider the subgroup generated by $a$ (or $b$, as this is the same subgroup). You have shown that the claim holds in this subgroup. How could it not hold in the bigger group?
$endgroup$
– Tobias Kildetoft
Dec 10 '18 at 8:22




1




1




$begingroup$
@Shawn. The problem arises in the case that the decomposition of $G$ into a product of cyclic $p$-groups contains two or more factors of order a power of $p$. In the case that $G$ is cyclic this can't happen, and one proves the assertion by first handling the case that $n$ is a prime power (which is easy) and then applying the Chinese Remainder Theorem.
$endgroup$
– falang
Dec 10 '18 at 8:26




$begingroup$
@Shawn. The problem arises in the case that the decomposition of $G$ into a product of cyclic $p$-groups contains two or more factors of order a power of $p$. In the case that $G$ is cyclic this can't happen, and one proves the assertion by first handling the case that $n$ is a prime power (which is easy) and then applying the Chinese Remainder Theorem.
$endgroup$
– falang
Dec 10 '18 at 8:26












$begingroup$
@Tobias: The bigger group may have order divisible by primes that do not divide the order of $a$.
$endgroup$
– falang
Dec 10 '18 at 8:29




$begingroup$
@Tobias: The bigger group may have order divisible by primes that do not divide the order of $a$.
$endgroup$
– falang
Dec 10 '18 at 8:29




1




1




$begingroup$
@Shaun: Where did I encounter the problem? I noticed that it was true for $G$ finite cyclic and wondered if it was true for arbitrary products of finite cyclic groups.
$endgroup$
– falang
Dec 10 '18 at 8:39




$begingroup$
@Shaun: Where did I encounter the problem? I noticed that it was true for $G$ finite cyclic and wondered if it was true for arbitrary products of finite cyclic groups.
$endgroup$
– falang
Dec 10 '18 at 8:39










1 Answer
1






active

oldest

votes


















0












$begingroup$

I should have thought more before posting... The answer is yes and the proof goes like this: Suppose first that $G$ is a $p$-group. By hypothesis there are integers $i$ and $j$ such that $ia=b$ and $jb=a$. We must show that it is possible to choose $i$ and $j$ prime to $p$. But the latter two equations imply that $ijb=b$, hence $(ij-1)b=0$. We may assume that $bne0$, otherwise there is nothing to prove. But then the order of $b$ divides $ij-1$. Since the order of $b$ is divisible by $p$, it follows that $i$ and $j$ are prime to $p$, which gives the required conclusion.



Finally, an arbitrary $G$ can be decomposed into a product of $p$-groups for distinct primes $p$. The result we want then follows readily from the Chinese Remainder Theorem.






share|cite|improve this answer









$endgroup$













    Your Answer





    StackExchange.ifUsing("editor", function () {
    return StackExchange.using("mathjaxEditing", function () {
    StackExchange.MarkdownEditor.creationCallbacks.add(function (editor, postfix) {
    StackExchange.mathjaxEditing.prepareWmdForMathJax(editor, postfix, [["$", "$"], ["\\(","\\)"]]);
    });
    });
    }, "mathjax-editing");

    StackExchange.ready(function() {
    var channelOptions = {
    tags: "".split(" "),
    id: "69"
    };
    initTagRenderer("".split(" "), "".split(" "), channelOptions);

    StackExchange.using("externalEditor", function() {
    // Have to fire editor after snippets, if snippets enabled
    if (StackExchange.settings.snippets.snippetsEnabled) {
    StackExchange.using("snippets", function() {
    createEditor();
    });
    }
    else {
    createEditor();
    }
    });

    function createEditor() {
    StackExchange.prepareEditor({
    heartbeatType: 'answer',
    autoActivateHeartbeat: false,
    convertImagesToLinks: true,
    noModals: true,
    showLowRepImageUploadWarning: true,
    reputationToPostImages: 10,
    bindNavPrevention: true,
    postfix: "",
    imageUploader: {
    brandingHtml: "Powered by u003ca class="icon-imgur-white" href="https://imgur.com/"u003eu003c/au003e",
    contentPolicyHtml: "User contributions licensed under u003ca href="https://creativecommons.org/licenses/by-sa/3.0/"u003ecc by-sa 3.0 with attribution requiredu003c/au003e u003ca href="https://stackoverflow.com/legal/content-policy"u003e(content policy)u003c/au003e",
    allowUrls: true
    },
    noCode: true, onDemand: true,
    discardSelector: ".discard-answer"
    ,immediatelyShowMarkdownHelp:true
    });


    }
    });














    draft saved

    draft discarded


















    StackExchange.ready(
    function () {
    StackExchange.openid.initPostLogin('.new-post-login', 'https%3a%2f%2fmath.stackexchange.com%2fquestions%2f3033611%2fthe-concept-of-associate-for-finite-abelian-groups%23new-answer', 'question_page');
    }
    );

    Post as a guest















    Required, but never shown

























    1 Answer
    1






    active

    oldest

    votes








    1 Answer
    1






    active

    oldest

    votes









    active

    oldest

    votes






    active

    oldest

    votes









    0












    $begingroup$

    I should have thought more before posting... The answer is yes and the proof goes like this: Suppose first that $G$ is a $p$-group. By hypothesis there are integers $i$ and $j$ such that $ia=b$ and $jb=a$. We must show that it is possible to choose $i$ and $j$ prime to $p$. But the latter two equations imply that $ijb=b$, hence $(ij-1)b=0$. We may assume that $bne0$, otherwise there is nothing to prove. But then the order of $b$ divides $ij-1$. Since the order of $b$ is divisible by $p$, it follows that $i$ and $j$ are prime to $p$, which gives the required conclusion.



    Finally, an arbitrary $G$ can be decomposed into a product of $p$-groups for distinct primes $p$. The result we want then follows readily from the Chinese Remainder Theorem.






    share|cite|improve this answer









    $endgroup$


















      0












      $begingroup$

      I should have thought more before posting... The answer is yes and the proof goes like this: Suppose first that $G$ is a $p$-group. By hypothesis there are integers $i$ and $j$ such that $ia=b$ and $jb=a$. We must show that it is possible to choose $i$ and $j$ prime to $p$. But the latter two equations imply that $ijb=b$, hence $(ij-1)b=0$. We may assume that $bne0$, otherwise there is nothing to prove. But then the order of $b$ divides $ij-1$. Since the order of $b$ is divisible by $p$, it follows that $i$ and $j$ are prime to $p$, which gives the required conclusion.



      Finally, an arbitrary $G$ can be decomposed into a product of $p$-groups for distinct primes $p$. The result we want then follows readily from the Chinese Remainder Theorem.






      share|cite|improve this answer









      $endgroup$
















        0












        0








        0





        $begingroup$

        I should have thought more before posting... The answer is yes and the proof goes like this: Suppose first that $G$ is a $p$-group. By hypothesis there are integers $i$ and $j$ such that $ia=b$ and $jb=a$. We must show that it is possible to choose $i$ and $j$ prime to $p$. But the latter two equations imply that $ijb=b$, hence $(ij-1)b=0$. We may assume that $bne0$, otherwise there is nothing to prove. But then the order of $b$ divides $ij-1$. Since the order of $b$ is divisible by $p$, it follows that $i$ and $j$ are prime to $p$, which gives the required conclusion.



        Finally, an arbitrary $G$ can be decomposed into a product of $p$-groups for distinct primes $p$. The result we want then follows readily from the Chinese Remainder Theorem.






        share|cite|improve this answer









        $endgroup$



        I should have thought more before posting... The answer is yes and the proof goes like this: Suppose first that $G$ is a $p$-group. By hypothesis there are integers $i$ and $j$ such that $ia=b$ and $jb=a$. We must show that it is possible to choose $i$ and $j$ prime to $p$. But the latter two equations imply that $ijb=b$, hence $(ij-1)b=0$. We may assume that $bne0$, otherwise there is nothing to prove. But then the order of $b$ divides $ij-1$. Since the order of $b$ is divisible by $p$, it follows that $i$ and $j$ are prime to $p$, which gives the required conclusion.



        Finally, an arbitrary $G$ can be decomposed into a product of $p$-groups for distinct primes $p$. The result we want then follows readily from the Chinese Remainder Theorem.







        share|cite|improve this answer












        share|cite|improve this answer



        share|cite|improve this answer










        answered Dec 10 '18 at 16:47









        SJRSJR

        51537




        51537






























            draft saved

            draft discarded




















































            Thanks for contributing an answer to Mathematics Stack Exchange!


            • Please be sure to answer the question. Provide details and share your research!

            But avoid



            • Asking for help, clarification, or responding to other answers.

            • Making statements based on opinion; back them up with references or personal experience.


            Use MathJax to format equations. MathJax reference.


            To learn more, see our tips on writing great answers.




            draft saved


            draft discarded














            StackExchange.ready(
            function () {
            StackExchange.openid.initPostLogin('.new-post-login', 'https%3a%2f%2fmath.stackexchange.com%2fquestions%2f3033611%2fthe-concept-of-associate-for-finite-abelian-groups%23new-answer', 'question_page');
            }
            );

            Post as a guest















            Required, but never shown





















































            Required, but never shown














            Required, but never shown












            Required, but never shown







            Required, but never shown

































            Required, but never shown














            Required, but never shown












            Required, but never shown







            Required, but never shown







            Popular posts from this blog

            Different font size/position of beamer's navigation symbols template's content depending on regular/plain...

            Berounka

            I want to find a topological embedding $f : X rightarrow Y$ and $g: Y rightarrow X$, yet $X$ is not...